Calculating Contour Integrals with Cauchy Theorem on Annulus/Donut Boundaries

  • Thread starter Thread starter mrjohns
  • Start date Start date
  • Tags Tags
    Cauchy Theorem
Click For Summary
The discussion centers on verifying Cauchy's theorem by calculating contour integrals for functions defined on an annulus. Participants express confusion about treating the annulus as a closed contour due to its inner and outer boundaries being disconnected. The suggested approach involves integrating over the outer boundary and subtracting the inner boundary, which effectively cancels contributions from a connecting line. Concerns are raised about handling the discontinuity at z=0 for the function f(z)=1/z, but it is clarified that the proposed method will not cause issues. Ultimately, the goal is to calculate the path integrals to confirm the theorem's validity.
mrjohns
Messages
13
Reaction score
0
A complex analysis question.

Homework Statement



Verify the Cauchy theorem by calculating the contour integrals.

Where ω is the appropriately orientated boundary of the annulus/donut defined by 1/3 ≤ IzI ≤ 2 for the following analytic functions:

i. f(z)=z^2
ii. f(z)=1/z

Homework Equations



closed ∫(ω) f(z)dz=0

closed ∫(ω) 1/(z-a) dz = 0 (a outside ω), 2∏i (a inside ω)

The Attempt at a Solution



The first bit that is confusing me is how ω can be considered a closed contour when it includes both the inner and outer boundaries of the annulus, which aren't connected. I'm not sure how it can be considered one closed contour and not two.

My plan of attack for the first one would be to just take the closed contour of the big circle, then subtract the little circle. Obviously both would be zero. But I'm not sure if this is the correct way to do things.

The second one obviously trying the same strategy would cause problems due to the discontiuity at z=0. So can I 'deform' the contours of each circle to exclude that central point? In that case I would get 2∏i - 2∏i = 0.

Is this the correct strategy or do I need to connect the outer and inner boundaries with a line?
 
Physics news on Phys.org
mrjohns said:
A complex analysis question.

Homework Statement



Verify the Cauchy theorem by calculating the contour integrals.

Where ω is the appropriately orientated boundary of the annulus/donut defined by 1/3 ≤ IzI ≤ 2 for the following analytic functions:

i. f(z)=z^2
ii. f(z)=1/z

Homework Equations



closed ∫(ω) f(z)dz=0

closed ∫(ω) 1/(z-a) dz = 0 (a outside ω), 2∏i (a inside ω)

The Attempt at a Solution



The first bit that is confusing me is how ω can be considered a closed contour when it includes both the inner and outer boundaries of the annulus, which aren't connected. I'm not sure how it can be considered one closed contour and not two.

My plan of attack for the first one would be to just take the closed contour of the big circle, then subtract the little circle. Obviously both would be zero. But I'm not sure if this is the correct way to do things.

The second one obviously trying the same strategy would cause problems due to the discontiuity at z=0. So can I 'deform' the contours of each circle to exclude that central point? In that case I would get 2∏i - 2∏i = 0.

Is this the correct strategy or do I need to connect the outer and inner boundaries with a line?

Yes, you need to connect the outer and inner boundaries with a line. But this is equivalent to taking the integral over the outer boundary and subtracting the integral over the inner boundary, because the contribution from the line cancels (it's traversed once in each direction).
 
Thanks. Does this mean I should really do it the first way you've said (rather than subtracting) as this would satisfy the lecturer better - or does it not really matter.

And if I do it that way, does that mean the f(z)=1/z case won't cause any issues anywhere?
 
mrjohns said:
Thanks. Does this mean I should really do it the first way you've said (rather than subtracting) as this would satisfy the lecturer better - or does it not really matter.

And if I do it that way, does that mean the f(z)=1/z case won't cause any issues anywhere?

The question does tell you to "verify Cauchy's theorem", so you have actually to calculate all the path integrals and show that their sum is the expected result.
 
Ok excellent, thanks for the help, will post if I have any issues.
 
Question: A clock's minute hand has length 4 and its hour hand has length 3. What is the distance between the tips at the moment when it is increasing most rapidly?(Putnam Exam Question) Answer: Making assumption that both the hands moves at constant angular velocities, the answer is ## \sqrt{7} .## But don't you think this assumption is somewhat doubtful and wrong?

Similar threads

  • · Replies 3 ·
Replies
3
Views
2K
  • · Replies 5 ·
Replies
5
Views
1K
Replies
1
Views
1K
Replies
32
Views
3K
  • · Replies 2 ·
Replies
2
Views
1K
  • · Replies 8 ·
Replies
8
Views
2K
  • · Replies 5 ·
Replies
5
Views
2K
Replies
4
Views
2K
  • · Replies 3 ·
Replies
3
Views
3K
  • · Replies 1 ·
Replies
1
Views
2K